Difference between revisions of "2010 AIME I Problems/Problem 12"

m (Solution)
 
(One intermediate revision by one other user not shown)
Line 7: Line 7:
 
We claim that <math>243</math> is the minimal value of <math>m</math>. Let the two partitioned sets be <math>A</math> and <math>B</math>; we will try to partition <math>3, 9, 27, 81,</math> and <math>243</math> such that the <math>ab=c</math> condition is not satisfied. [[Without loss of generality]], we place <math>3</math> in <math>A</math>. Then <math>9</math> must be placed in <math>B</math>, so <math>81</math> must be placed in <math>A</math>, and <math>27</math> must be placed in <math>B</math>. Then <math>243</math> cannot be placed in any set, so we know <math>m</math> is less than or equal to <math>243</math>.
 
We claim that <math>243</math> is the minimal value of <math>m</math>. Let the two partitioned sets be <math>A</math> and <math>B</math>; we will try to partition <math>3, 9, 27, 81,</math> and <math>243</math> such that the <math>ab=c</math> condition is not satisfied. [[Without loss of generality]], we place <math>3</math> in <math>A</math>. Then <math>9</math> must be placed in <math>B</math>, so <math>81</math> must be placed in <math>A</math>, and <math>27</math> must be placed in <math>B</math>. Then <math>243</math> cannot be placed in any set, so we know <math>m</math> is less than or equal to <math>243</math>.
  
For <math>m \le 242</math>, we can partition <math>S</math> into <math>S \cap \{3, 4, 5, 6, 7, 8, 81, 82, 83, 84 ... 242\}</math> and <math>S \cap \{9, 10, 11 ... 80\}</math>, and in neither set are there values where <math>ab=c</math> (since <math>8 < (3\text{ to }8)^2 < 81</math> and <math>(9\text{ to }80)^2 > 80 </math>). Thus <math>m = \boxed{243}</math>.
+
For <math>m \le 242</math>, we can partition <math>S</math> into <math>S \cap \{3, 4, 5, 6, 7, 8, 81, 82, 83, 84 ... 242\}</math> and <math>S \cap \{9, 10, 11 ... 80\}</math>, and in neither set are there values where <math>ab=c</math> (since <math>8 < (3\text{ to }8)^2 < 81</math> and <math>81^2>242</math> and <math>(9\text{ to }80)^2 > 80 </math>). Thus <math>m = \boxed{243}</math>.
  
 
==Video Solution==
 
==Video Solution==
  
https://youtu.be/DNC2zd5rReY
+
https://youtu.be/WCPxL5dLKCI
  
 
~Shreyas S
 
~Shreyas S

Latest revision as of 21:56, 25 November 2023

Problem

Let $m \ge 3$ be an integer and let $S = \{3,4,5,\ldots,m\}$. Find the smallest value of $m$ such that for every partition of $S$ into two subsets, at least one of the subsets contains integers $a$, $b$, and $c$ (not necessarily distinct) such that $ab = c$.

Note: a partition of $S$ is a pair of sets $A$, $B$ such that $A \cap B = \emptyset$, $A \cup B = S$.

Solution

We claim that $243$ is the minimal value of $m$. Let the two partitioned sets be $A$ and $B$; we will try to partition $3, 9, 27, 81,$ and $243$ such that the $ab=c$ condition is not satisfied. Without loss of generality, we place $3$ in $A$. Then $9$ must be placed in $B$, so $81$ must be placed in $A$, and $27$ must be placed in $B$. Then $243$ cannot be placed in any set, so we know $m$ is less than or equal to $243$.

For $m \le 242$, we can partition $S$ into $S \cap \{3, 4, 5, 6, 7, 8, 81, 82, 83, 84 ... 242\}$ and $S \cap \{9, 10, 11 ... 80\}$, and in neither set are there values where $ab=c$ (since $8 < (3\text{ to }8)^2 < 81$ and $81^2>242$ and $(9\text{ to }80)^2 > 80$). Thus $m = \boxed{243}$.

Video Solution

https://youtu.be/WCPxL5dLKCI

~Shreyas S

See Also

2010 AIME I (ProblemsAnswer KeyResources)
Preceded by
Problem 11
Followed by
Problem 13
1 2 3 4 5 6 7 8 9 10 11 12 13 14 15
All AIME Problems and Solutions

The problems on this page are copyrighted by the Mathematical Association of America's American Mathematics Competitions. AMC logo.png